Criminal Law Flashcards

1
Q

A woman who is a computer whiz decided to dedicate herself to exposing person who traffic in CP. She posed a number of sexually oriented photos on her web site. The file for each photo contained an embedded Trojan horse program. The D downloaded one of those photos onto his PC. Using the embedded program, the woman entered the D’s computer and found a file containing a pornographic photo of a child. She copied the file and turned it over to a federal law enforcement agency.

A federal agent told her that a successful prosecution would require more than one photo and offered here a monetary reward for additional photos leading to a conviction of the D. The woman entered the D’s computer again, and this time she phoned hundreds of CP photos, which she turned over to the federal agency.

The D is charged with multiple counts of violating federal statutes regarding CP. He moves to suppress the photos that the woman discovered on his computer. The motion is based on both the 4th Amendment and a federal statute forbidding interception of electronic communications whiteout permission. The parties have stipulated that the woman’s conduct in downloading photos from the D’s computer violated the interception statute.

How should the court rule on the defendant’s motion to suppress?

(A) Grant it as to all photographs, because the woman violated the federal interception statute.
(B) Grant it as to all photographs, because the woman acted without probable cause.
(C) Grant it only to the second set of photographs.
(D) Deny it as to all photographs.

A

C

How well did you know this?
1
Not at all
2
3
4
5
Perfectly
2
Q

The police suspected a woman of growing marijuana in her private residence. Narcotics officers went to her neighborhood in the middle of the night. Nothing unlawful could be seen from the street, so the officers walked into the neighbors’ yard and looked through the woman’s kitchen window, which had neither drapes nor shades. The officers observed what appeared to be marijuana plants being cultivated under grow lights in the kitchen. Using this information, the officers obtained a search warrant. The execuation of that warrant netted numerous marijuana plants.

The woman was charged with possession of marijuana. She moved to suppress the marijuana plants recovered when the warrant was executed, claiming that the evidence supportiing the warrant was obtained through a search that violated the 4th Amendment.

Should the marijuana plants be suppressed?

(A) No, because regardless of the lawfulness of the police conduct beforehand, they did obtain a warrant to search the woman’s home.
(B) No, because the woman could have no reasonable expectation of privacy concerning activities that she expose to the view of her neighbors.
(C) Yes, because the officers’ clandestine observation of the plants violated the woman’s reasonable expectation of privacy concerning activities occurring in her home.
(D) Yes, because no unlawful activities could be observed by the officers from any public vantage point.

A

B

How well did you know this?
1
Not at all
2
3
4
5
Perfectly
3
Q

US customs officials received an anonymous tip that heroin would be found inside a distinctively marked red package mailed from a foreign country to the US. Pursuant to this tip, US customs officers opened the red package and found heroin inside. They then resealed the package and left the heroin inside it. The FBI was notified and, as agents watched, the package was delivered to the address.

The FBI then secured a warrant to search the house for the package. About two hours after the package was delivered, the warrant was executed at the house. The man who opened the door was arrested, and the agents found the package, unopened, in an upstairs bedroom closet. After seizing the package, the agents looked through the rest of the house. In a footlocker in the basement, they found a machine gun.

The man was charged with, among other things, unlawful possession of the machine gun. He moved to suppress the evidence.

Should the court grant the motion to suppress the machine gun?

(A) Yes, because the search exceeded the authority granted by the warrant.
(B) Yes, because the initial search by the customs officers was without probable cause.
(C) No, because having found the package, the agents had probable cause to believe more narcotics could be located in the house and the gun was found in a proper search for narcotics.
(D) No, because narcotics dealers are often armed and the search was justified to protect the officers.

A

A

How well did you know this?
1
Not at all
2
3
4
5
Perfectly
4
Q

Police officers received a tip that drug dealing was occurring at a certain ground-floor duplex apartment. They decided to stake out the apartment. The stakeout revealed that a significant number of people visited the apartment for short periods of time and then left. A man exited the apartment and started to walk away. the officers grabbed the man and, when he struggled, wrestled him to the ground. They searched him and found a bag of heroin in one of his pockets. After discovering the heroin on the man, the police decided to enter the apartment. They knocked on the door, which was opened by the woman who livd there. The police asked if they could come inside, and the woman gave them permission to do so. Once inside, the officers observed several bags of heroin on the living room table. The woman is charged with possession of the heroin found on the living room table. She moves pretrial to suppress the heroin on the ground that it was obtained by virtue of an illegal search and seizure.

Should the motion be granted?

(A) No, because the tip together with the heroin found in the man’s pocket provided probable cause for the search
(B) No, because the woman consented to the officers’ entry
(C) Yes, because the officers’ decision to enter the house was the fruit of an illegal search of the man
(D) Yes, because the officers did no inform the woman that she could refuse consent.

A

B

How well did you know this?
1
Not at all
2
3
4
5
Perfectly
5
Q

FBI agents, without a warrant and without permission of Mexican law enforcement or judicial officers, entered Mexico, kidnapped S, an American citizen wanted the US for drug smuggling violations, and forcibly drove him back to Texas. Thereafter, the agents, again without a warrant, broke into the Texas home of J, wanted as a confederate of S, and arrested her. S and J were both indicted for narcotics violations. Both moved to dismiss the indictment on the ground that their arrests violated the 4th Amendment.

The court should

(A) grant the motions of both S and J.
(B) grant the motion of J and deny the motion of S.
(C) deny the motions of both S and J
(D) grant the motions of S and deny the motion of J.

A

C

How well did you know this?
1
Not at all
2
3
4
5
Perfectly
6
Q

Police, who had probable cause to arrest a man for a series of armed robberies, obtained a warrant to arrest him. At 6 am, they surreptitiously entered the Mans’ house and, with guns drawn, went to the mans’ bedroom where they awakened him. Started, the man asked, “What’s going on?” and an officer replied, “We’ve got you now.” Another officer immediately asked the man if he had committed a particular robbery, and the man said that he had. The police then informed him that he was under arrest and ordered him to get dressed. Charged with robbery, the man has moved to suppress the use of his statement as evidence.

What is the man’s best argument for granting his motion?

(A) The police did not read him the required Miranda warnings.
(B) He was not informed that he was under arrest until after he made the statement.
(C) The statement was not voluntary.
(D) The police did not have a search warrant authorizing entry into the house.

A

A

How well did you know this?
1
Not at all
2
3
4
5
Perfectly
7
Q

In a city, a number of armed bank robberies were committed near closing time by a masked man wearing a white hooded sweatshirt and blue sweatpants. Police saw a man wearing a white hooded sweatshirt and blue sweatpants pacing nervously outside one of the city’s banks just beefre it closed. The police stopped the man and frisked the outer layers of his clothing for weapons but found none. They asked the man what he was doing outside the bank and pointed out that he ws wearing clothing similar to clothing worn by the perpetrator of recent robberies. After pausing several moments, the man confessed. The police had not provided him with any Miranda warnings. After being charged with the bank robberies, the man moved to suppress his confession. The parties agreed, and the court properly found, that the police had reasonable suspicion but no probable cause at all times before the man confessed.

Should the motion to suppress be granted?

(A) Yes, because the confession was the fruit of a 4th Amendment violation, even though there was no Miranda warnings.
(B) Yes, because the confession was the fruit of both a 4th Amendment violation and a Miranda violation.
(C) Yes, because the confession was the fruit of a Miranda violation, even though there was no Miranda violation.
(D) No, because there was neither a 4th Amendment no a Miranda violation.

A

D

How well did you know this?
1
Not at all
2
3
4
5
Perfectly
8
Q

A Defendant was lawfully arrested without a warrant for bank robbery. He was not given Miranda warnings, but was immediately taken to a police station where he and five other men were placed in a lineup to be viewed by the bank teller. Each man was required to say the words spoken by the bank robber: “Give me all your money, I’ve got a gun.” After all the men in the lineup spoke those words, the teller identified the defendant as the robber. The defendant subsequently moved to suppress the testimony of the teller, claiming the lineup violated his privilege against self-incrimination. At a suppression hearing, the teller testified that she had not gotten a good look at the robber’s face, because the robber had been wearing a hat pulled down over most of his face, but that she was certain that the defendant was the robber because she had recognized his voice at the lineup.

Should the D’s motion be granted?

(A) No, because being required to speak at the lineup, while compelled, was not testimonial or communicative.
(B) No, because the testimony of a witness based on firsthand observations is not subject to exclusion as the fruit of the poisonous tree.
(C) Yes, because the defendant was compelled to speak at the lineup, and this compelled speech led to the witness’s identification testimony.
(D) Yes, because the defendant was never informed that he could refuse to make a statement and that any statement could be used as evidence against him.

A

A

How well did you know this?
1
Not at all
2
3
4
5
Perfectly
9
Q

A store owner whose jewelry store had recently been robbed was shown by a police detective a photograph of the defendant, who previously had committed other similar crimes. The store owner examined the photo and then asked the detective whether the police believed that the man pictures was the robber. After the detective said, “We’re pretty sure,” the store owner stated that the man in the photo was the one who had robbed her. The defendant was indicted for the robbery. His counsel moved to suppress any trial testimony by the store owner identifying the defendant as the robber.

Should the court grant the motion and suppress the store owners trial testimony identifying the defendant as the robber?

(A) No, because the out-of-court identification was not improper.
(B) No, because suppression of in-court testimony is not a property remedy, even though the out-of-court identification was improper.
(C) Yes, because the improper out-of-court identification has necessarily tainted any in-court identification.
(D) Yes, unless the prosecution demonstrates that the in-court identification is reliable.

A

D

How well did you know this?
1
Not at all
2
3
4
5
Perfectly
10
Q

A Defendant was charged with manslaughter. At the preliminary hearing, the magistrate dismissed the charge on the grounds that the evidence was insufficient. The prosecutor then brought the case before a grand jury. After hearing the evidence presented by the prosecutor, the grand jury refused to return and indictment. The prosecutor waited a few months until a new grand jury had been impaneled and brought the case before that grand jury, which returned an indictment charging the defendant with manslaughter. The defendant moves to dismiss the indictment on double jeopardy grounds.

Should the motion be granted?

(A) No, because jeopardy had not attached.
(B) No, because there has been no conviction or acquittal.
(C) Yes, because any proceedings after the preliminary hearing would violate double jeopardy.
(D) Yes, because brining the case before the second grand jury was a violation of double jeopardy.

A

A

How well did you know this?
1
Not at all
2
3
4
5
Perfectly